2018 AMC 12A Problem #7

For how many (not necessarily positive) integer values of n n is the value of 4000 ( 2 5 ) n 4000 \cdot \left ( \frac{2}{5} \right )^n an integer?


Next problem

Previous problem

All problems

3 9 8 6 4

This section requires Javascript.
You are seeing this because something didn't load right. We suggest you, (a) try refreshing the page, (b) enabling javascript if it is disabled on your browser and, finally, (c) loading the non-javascript version of this page . We're sorry about the hassle.

2 solutions

Zain Majumder
Feb 11, 2018

2 2 and 5 5 are coprime, so the numerator will not affect whether the expression is an integer. For example, no matter how many times a number is doubled, it will not change whether the product is divisible by 5 5 . Therefore, we will only consider the denominator.

If n n is positive, the denominator will be a power of 5 5 . If n n is negative, the denominator will be a power of 2 2 . 4000 = 2 5 5 3 4000 = 2^5 \cdot 5^3 , so n n may not exceed these exponents. Therefore, 5 n 3 -5 \le n \le 3 (including 0 0 ), so there are 3 ( 5 ) + 1 = 9 3-(-5)+1 = \boxed{9} possible values of n n .

Jerry McKenzie
Feb 9, 2018

Note for 4000 ( 2 5 ) n 4000(\frac{2}{5})^{n} the denominator of ( 2 5 ) n (\frac{2}{5})^{n} must cancel with 4000.

Trivially we get n = 0 4000 ( 2 5 ) n = 4000 n=0 \Rightarrow 4000(\frac{2}{5})^{n}=4000

Since 4000 = 2 5 5 3 4000=2^{5}\cdot 5^{3} then if n is positive, then 5 n 5^n must cancel with 5 3 5^3 , it follows n 3 [ [ n N ] ] n\leq 3 \enspace [[n\in\mathbb{N}]] .

If n is negative, then 2 n 2^n must cancel with 2 5 2^5 , it follows n 5 [ [ n N ] ] n\geq -5 \enspace [[n\in\mathbb{N}]] .

Note if n is not an integer, ( 2 5 ) n (\frac{2}{5})^{n} Is irrational.

Thus 5 n 3 [ [ n N ] ] -5\leq n\leq 3 \enspace [[n\in\mathbb{N}]] .

With 3 ( 5 ) + 1 = 9 3-(-5)+1=9 solutions.

0 pending reports

×

Problem Loading...

Note Loading...

Set Loading...